Lösung einer Gleichung mit komplexen Zahlen

Neue Frage »

Andy0810 Auf diesen Beitrag antworten »
Lösung einer Gleichung mit komplexen Zahlen
Meine Frage:
Ich habe in der Uni folgende Aufgabe gestellt bekommen:
Bestimmen sie alle Lösungen z|C der Gleichung
16-=i

Prüfen sie, ob eine Lösung z mit Re(z)<0 und Im(z)>0 existiert.
Hinweis: Es gilt cos}{6}[/latex]={2}}[/latex] und sin}{6}[/latex]=.

Meine Ideen:
Ich habe in der Gleichung zunächst -i gerechnet:
16--i=0
Dann weiß ich noch, dass folgendes gilt:
=*(cos(n)+i*sin(n))=*}[/latex]
jetzt weiß ich aber nicht mehr weiter. Ich weiß das es 3 Lösungen geben muss wegen . Aber welche Lösungen, und wie ich darauf komme, weiß ich nicht. Kann mir da jemand weiterhelfen?

lg Andy
Airblader Auf diesen Beitrag antworten »
RE: Lösung einer Gleichung mit komplexen Zahlen
Den Vorschau-Button gibt es nicht nur zum Anschauen und Ignorieren. Aber da ich einen guten Tag habe:

Zitat:
Original von Andy0810
Meine Frage:
Ich habe in der Uni folgende Aufgabe gestellt bekommen:

Bestimmen Sie alle Lösungen der Gleichung

Prüfen Sie, ob eine Lösung mit und existiert.

Hinweis: Es gilt und .

Meine Ideen:
Ich habe in der Gleichung zunächst -i gerechnet:

Dann weiß ich noch, dass folgendes gilt:


Jetzt weiß ich aber nicht mehr weiter. Ich weiß, dass es 3 Lösungen geben muss wegen . Aber welche Lösungen, und wie ich darauf komme, weiß ich nicht. Kann mir da jemand weiterhelfen?

lg Andy


air
Steffen Bühler Auf diesen Beitrag antworten »
RE: Lösung einer Gleichung mit komplexen Zahlen
Zitat:
Original von Andy0810
Ich habe in der Gleichung zunächst -i gerechnet:
16--i=0


Einfacher wird's, wenn Du



hinschreibst. Und dann am besten rechts polar ausdrücken.

Viele Grüße
Steffen
Andy0810 Auf diesen Beitrag antworten »
RE: Lösung einer Gleichung mit komplexen Zahlen
Danke, ich hab vor jeden mathematischen Begriff latex geschrieben. Anscheinend soll man das nur 1 mal und nicht für jeden. Vielen Dank für die Verbesserung
ollie3 Auf diesen Beitrag antworten »
RE: Lösung einer Gleichung mit komplexen Zahlen
hallo andy,
also erstmal ein technischer hinweis: wenn du latex benutzt, reicht es, wenn
man nur am anfang und am ende der betreffende zeile latex bzw. /latex
schreibt, also nicht mehrmals latex ineinander verschachteln.
Und zu der aufgabe, du musst die gleichung nach z^3 auflösen, und dann kannst
du tatsächlich deine lösungsformel anwenden, um die 3 lösungen zu bekommen.
gruss ollie3
Andy0810 Auf diesen Beitrag antworten »
RE: Lösung einer Gleichung mit komplexen Zahlen
Ich habe dann folgende Gleichung:



also + danach Polardarstellung und durch 16 geteilt

Ich hab das Gefühl, dass es eigentlich schon kurz vor dem Ziel ist. Aber ich komme nicht auf den nächsten Schritt. Ich könnte die 3te Wurzel ziehen, aber dann wirds nicht unbedingt schöner. Auch weiß ich nicht was ich mit dem anfangen soll.
 
 
original Auf diesen Beitrag antworten »
RE: Lösung einer Gleichung mit komplexen Zahlen
Zitat:
Original von Andy0810
Ich habe dann folgende Gleichung:

Gott


du sollst doch



nach z^3 = auflösen ?! versuch das nochmal..

und stelle dann



in (zB trigonometrischer) Polarform dar...

also:

sorry, Steffen Bühler , dachte du bist nicht on...
Steffen Bühler Auf diesen Beitrag antworten »
RE: Lösung einer Gleichung mit komplexen Zahlen
Da stimmt was nicht.

Wie berechnest Du die Polardarstellung von ?

Viele Grüße
Steffen
Andy0810 Auf diesen Beitrag antworten »

Vielen Dank für Antworten!
Ich habe die Formel benutzt
wobei n=1 ist. Ich habe als z gewählt und angewendet.
da beim Imaginärteil kein Vorfaktor ist.

Ich habe dann noch auf beiden Seiten durch 16 geteilt und bin dann auf die Gleichung gekommen. Ist aber anscheinend irgendwo ein Denkfehler drin.

Wenn ich direkt nach auflöse, komme ich auf folgendes:




Ich würde es dann so umstellen:





Ich meine mich zu erinnern, dann man für auch arctan(b) schreiben kann (b aus z=a+ib). Das wäre dann
Bin mir dabei aber nicht sicher.
Sollte das funktionieren würde dann stehen:



und schon weiß ich nicht mehr weiter. Ich weiß, man kann es irgendwie mit dem Einheitskreis lösen, bzw man geht im Einheitskreis um gewissen Winkel weiter. Aber ich bekomme den Zusammenhang nicht mehr hin.

Viele Grüße und Dank
Andy
Steffen Bühler Auf diesen Beitrag antworten »

Zitat:
Original von Andy0810
Ich habe als z gewählt und angewendet.
da beim Imaginärteil kein Vorfaktor ist.


Oh. Da ist durchaus ein Vorfaktor, nämlich Eins.

Stell Dir's grafisch vor: das ist die Zahl in der xy-Ebene mit und . Die Länge dieses Zeigers ist was? Und der Winkel?

Viele Grüße
Steffen
Andy0810 Auf diesen Beitrag antworten »

Aha. Ich erkenne den Fehler.
die Formel besagt ja

Damit wäre es




Die Länge des Zeigers wäre |z| und der Winkel müsste den Vektor von |z| mit der x-Achse einschließen.
Steffen Bühler Auf diesen Beitrag antworten »

Zitat:
Original von Andy0810


Die Länge des Zeigers wäre |z|


Richtig!

Zitat:
und der Winkel müsste den Vektor von |z| mit der x-Achse einschließen.


Ebenfalls richtig. Und hier kommt dann tatsächlich der Arkustangens dazu:



Viele Grüße
Steffen
Andy0810 Auf diesen Beitrag antworten »

Zitat:
Original von Steffen Bühler
Zitat:
Original von Andy0810


Die Länge des Zeigers wäre |z|


Richtig!

Zitat:
und der Winkel müsste den Vektor von |z| mit der x-Achse einschließen.


Ebenfalls richtig. Und hier kommt dann tatsächlich der Arkustangens dazu:



Viele Grüße
Steffen


Dann weiß ich jetzt eine Lösung?
Ich verstehe gerade den Zusammenhang nicht.

Liebe Grüße
Andy
Steffen Bühler Auf diesen Beitrag antworten »

Es geht uns um



Wenn wir Betrag und Winkel der rechten Seite kennen, dividieren wir noch auf beiden Seiten durch 16. Dann dividiert sich der Betrag, der Winkel bleibt gleich.

So. Jetzt wollen wir daraus die dritte Wurzel ziehen. Da ziehst Du die dritte Wurzel des Betrags und dividierst den Winkel durch drei.

Jetzt haben wir die Hauptlösung. Aber wie Du ja geschrieben hast, gibt es drei Lösungen, die sich gleichmäßig auf dem Betragskreis verteilen. Hier also im 120-Grad-Abstand.

Was bekommst Du dann raus?

Viele Grüße
Steffen
Andy0810 Auf diesen Beitrag antworten »

Ich stehe gerade völlig auf dem Schlauch.

Ich habe das so verstanden








muss ich das jz mit 1/3 , 2/3 und 3/3 multiplizieren?
Steffen Bühler Auf diesen Beitrag antworten »

Zitat:
Original von Andy0810







Die 2 hast Du ja schon richtig berechnet, deshalb hab ich sie mal eingesetzt.

Nun zum Winkel:





Viele Grüße
Steffen
Andy0810 Auf diesen Beitrag antworten »

Vielen Dank!
Ich hab jetzt auch verstanden was es mit dem Winkel auf sich hat.
Ich vermute ich muss nun folgendes machen:


wobei
ich würde dann noch eine Variable k einführen, um den Einheitskreis "abzulaufen"
eine Runde um den Einheitskreis ist und die 3, weil der Exponent bei z 3 ist. k={0,1,2}

Dann wäre die Gleichung:





Stimmt das?

Liebe Grüße
Andy
Steffen Bühler Auf diesen Beitrag antworten »

Zitat:
Original von Andy0810



Nicht so schnell. Das mit dem k kommt erst später. Also:



Aber so stimmt's jetzt.

Und nun, wie Schreiber dieses bereits weiter oben erwähnt hat:

Zitat:
Jetzt wollen wir daraus die dritte Wurzel ziehen. Da ziehst Du die dritte Wurzel des Betrags und dividierst den Winkel durch drei.


Na?

Viele Grüße
Steffen
Andy0810 Auf diesen Beitrag antworten »

Dann wäre die Gleichung:


und

Damit dürfte die Gleichung dann so aussehen:


Jetzt sollte aber die Variable eingeführt werden, oder?

Zitat:
ich würde dann noch eine Variable k einführen, um den Einheitskreis "abzulaufen" eine Runde um den Einheitskreis ist und die 3, weil der Exponent bei z 3 ist. k={0,1,2}


Liebe Grüße
Andy
Steffen Bühler Auf diesen Beitrag antworten »

Zitat:
Original von Andy0810



Na, wunderbar! Die Hauptlösung!

Zitat:
Jetzt sollte aber die Variable eingeführt werden, oder?


Ja, oder Du schreibst die zwei anderen Lösungen einfach dazu. Finde ich persönlich lesbarer.

Viele Grüße
Steffen
Andy0810 Auf diesen Beitrag antworten »

Wie kann ich das begründen, dass die anderen beiden auch eine Lösung sind, ohne das ich die Variable einführe?

Vielen Vielen Vielen Dank, du hast mich gerettet.
Steffen Bühler Auf diesen Beitrag antworten »

Zitat:
Original von Andy0810
Wie kann ich das begründen, dass die anderen beiden auch eine Lösung sind, ohne das ich die Variable einführe?


Mußt Du bei x²=4 begründen, daß auch -2 eine Lösung ist?

Wenn Du's aber gerne zeigen willst, nimm die einzelnen Lösungen jeweils hoch drei.

Viele Grüße
Steffen
Andy0810 Auf diesen Beitrag antworten »

Stimmt auch wieder.
Jetzt bleibt nur noch der zweite Aufgabenteil:
Prüfen sie, ob eine Lösung z mit Re(z)<0 und Im(z)>0 existiert.
Hinweis: Es gilt

Ich würde jz die Formel umstellen in:

und alle 3 Lösungen ausrechnen. Dann ist entweder eine dabei die die Bedingung erfüllt oder nicht, wobei ich davon ausgehe, dass keine dabei ist.
JuRo Auf diesen Beitrag antworten »

doch, wenn man sich die drei Lösungen anzeigen lässt, gibt es einen Punkt im 2. Quadranten ( Re(z)<0, Im(z)>0 ).
Aber ich glaube dein Lösungsansatz zum zweiten Teil stimmt.
Werde das mal ausprobieren

Bin übrigens gleiche Uni, gleiches Semester, muss die Aufgabe auch machen Augenzwinkern
Steffen Bühler Auf diesen Beitrag antworten »

Zitat:
Original von Andy0810
Ich würde jz die Formel umstellen in:
...
und alle 3 Lösungen ausrechnen.


Mach Dir das Leben nicht so schwer. Wenn die Winkel und für Dich unanschaulich sind (mir geht's jedenfalls so), nenn sie einfach 10° und 120°. Die Hauptlösung zeigt also auf 10°, für die anderen zwei drehst Du eben 120° weiter. Das sieht dann aus wie ein Mercedes-Stern. Und dann siehst Du eben auch sofort, um welchen Punkt es geht.

Zitat:
Original von JuRo
doch, wenn man sich die drei Lösungen anzeigen lässt, gibt es einen Punkt im 2. Quadranten ( Re(z)<0, Im(z)>0 ).


Recht hast Du. Nur das mit dem "anzeigen lassen" ist nicht nötig.

Zitat:
Original von JuRo
Aber ich glaube dein Lösungsansatz zum zweiten Teil stimmt.


Er stimmt ja auch, ist nur ein wenig umständlich. Was, wenn es zweiundvierzig Lösungen gibt? Auch alle "anzeigen lassen" oder über Cosinus und Sinus ausrechnen?

Ich muß mal wieder meinen Lehrer aus dem 5. Schuljahr zitieren: "Mathematiker sind furchtbar faul." Ich halte mich heute noch dran.

Viele Grüße
Steffen
Neue Frage »
Antworten »



Verwandte Themen

Die Beliebtesten »
Die Größten »
Die Neuesten »